"Electromagnetics"의 두 판 사이의 차이

수학노트
둘러보기로 가기 검색하러 가기
imported>Pythagoras0
imported>Pythagoras0
30번째 줄: 30번째 줄:
 
==Lagrangian formulation==
 
==Lagrangian formulation==
  
*  Lagrangian for a charged particle in an electromagnetic field<br><math>L=T-V</math><br><math>L(q,\dot{q})=m||\dot{q}||-e\phi+eA_{i}\dot{q}^{i}</math><br>
+
*  Lagrangian for a charged particle in an electromagnetic field <math>L=T-V</math>
*  action<br><math>S=-\frac{1}{4}\int F^{\alpha\beta}F_{\alpha\beta}\,d^{4}x</math><br>
+
:<math>L(q,\dot{q})=m||\dot{q}||-e\phi+eA_{i}\dot{q}^{i}</math><br>
*  Euler-Lagrange equations<br><math>p_{i}=\frac{\partial{L}}{\partial{\dot{q}^{i}}}=m\frac{\dot{q}_{i}}{||\dot{q}_{i}||}+eA_{i}=mv_{i}+eA_{i}</math><br><math>F_{i}=\frac{\partial{L}}{\partial{{q}^{i}}}=\frac{\partial}{\partial{{q}^{i}}}(eA_{j}\dot{q}^{j})=e\frac{\partial{A_{j}}}{\partial{q}^{i}}\dot{q}^{j}}}</math><br>
+
*  action
*  equation of motion<br><math>\dot{p}=F</math> Therefore we get<br><math>m\frac{dv_{i}}{dt}=eF_{ij}\dot{q}^{j}</math>. This is what we call the Lorentz force law.<br>
+
:<math>S=-\frac{1}{4}\int F^{\alpha\beta}F_{\alpha\beta}\,d^{4}x</math><br>
 +
*  Euler-Lagrange equations
 +
:<math>p_{i}=\frac{\partial{L}}{\partial{\dot{q}^{i}}}=m\frac{\dot{q}_{i}}{||\dot{q}_{i}||}+eA_{i}=mv_{i}+eA_{i}</math>
 +
$$
 +
F_{i}=\frac{\partial{L}}{\partial{q^{i}}}=\frac{\partial}{\partial{{q}^{i}}}(eA_{j}\dot{q}^{j})=e\frac{\partial{A_{j}}}{\partial{q}^{i}}\dot{q}^{j}
 +
$$
 +
*  equation of motion<br><math>\dot{p}=F</math> Therefore we get
 +
:<math>m\frac{dv_{i}}{dt}=eF_{ij}\dot{q}^{j}</math>. This is what we call the Lorentz force law.<br>
 
* force on a particle is same as <math>e\mathbf{E}+e\mathbf{v}\times \mathbf{B}</math>
 
* force on a particle is same as <math>e\mathbf{E}+e\mathbf{v}\times \mathbf{B}</math>
  

2013년 3월 23일 (토) 04:52 판

gauge invariance

  • the electromagnetic potential is a connection on a U(1)-bundle on spacetime whose curvature is the electromagnetic field
  • the electromagnetism is a gauge field theory with structure group U(1)

 

 

Lorentz force

  • almost all forces in mechanics are conservative forces, those that are functions only of positions, and certainly not functions of velocities
  • Lorentz force is a rare example of velocity dependent force

 

 

polarization of light

  • has two possibilites
    • what does this mean?

 

 

 

Lagrangian formulation

  • Lagrangian for a charged particle in an electromagnetic field \(L=T-V\)

\[L(q,\dot{q})=m||\dot{q}||-e\phi+eA_{i}\dot{q}^{i}\]

  • action

\[S=-\frac{1}{4}\int F^{\alpha\beta}F_{\alpha\beta}\,d^{4}x\]

  • Euler-Lagrange equations

\[p_{i}=\frac{\partial{L}}{\partial{\dot{q}^{i}}}=m\frac{\dot{q}_{i}}{||\dot{q}_{i}||}+eA_{i}=mv_{i}+eA_{i}\] $$ F_{i}=\frac{\partial{L}}{\partial{q^{i}}}=\frac{\partial}{\partial{{q}^{i}}}(eA_{j}\dot{q}^{j})=e\frac{\partial{A_{j}}}{\partial{q}^{i}}\dot{q}^{j} $$

  • equation of motion
    \(\dot{p}=F\) Therefore we get

\[m\frac{dv_{i}}{dt}=eF_{ij}\dot{q}^{j}\]. This is what we call the Lorentz force law.

  • force on a particle is same as \(e\mathbf{E}+e\mathbf{v}\times \mathbf{B}\)

 

 

Hamiltonian formulation

  • total energy of a charge particle in an electromagnetic field
    \(E=\frac{1}{2m}(p_j-eA_{j})(p_j-eA_j)+q\phi\)
  • replace the momentum with the canonical momentum
    • similar to covariant derivative

 

 

force on a particle

  • force on a particle is same as \(e\mathbf{E}+e\mathbf{v}\times \mathbf{B}\)

 

 

 

 

메모

 

 

related items

 

 

encyclopedia

 

 

books

ELECTROMAGNETIC THEORY AND COMPUTATION